LSAT 46 – Section 3 – Question 11

You need a full course to see this video. Enroll now and get started in less than a minute.

Target time: 0:57

This is question data from the 7Sage LSAT Scorer. You can score your LSATs, track your results, and analyze your performance with pretty charts and vital statistics - all with a Free Account ← sign up in less than 10 seconds

Question
QuickView
Type Tags Answer
Choices
Curve Question
Difficulty
Psg/Game/S
Difficulty
Explanation
PT46 S3 Q11
+LR
Must be true +MBT
A
2%
151
B
95%
164
C
1%
148
D
2%
156
E
0%
148
132
139
146
+Easier 145.195 +SubsectionEasier

We have an MBT question which we can glean from the question stem which reads: If the statements above are true, then on the basis of them which one of the following must also be true?

Our stimulus tells us that in the year 1990, the municipality of Queesnton raised taxes that increased the budget of its school system. The schools in turn used the increase in budget to increase the number of teachers they employed by 30%. However, the average number of students per teacher remained constant between 1990 and 1993.

This is almost phrased like an RRE question, right? It’s constructed as if there’s a paradox here. But let’s think about this: is it hard to reconcile the fact that the number of teachers went up while the average number of students to teachers stayed the same? No! Think about it: if the total number of dogs went up in NYC but the number of dogs per household stayed the same, would that make sense? Yes! It just means there are more households that own dogs. If we think about this as a fraction, both the numerator and denominator (top and bottom) of the fraction went up at the same rate. The same thing could be true for our students per teacher average, right? If the number of teachers went up and the number of students rose at the same rate (in this case 30%), then the average number of students per teacher would remain the same.

Ok now that we’ve synthesized the information here, let’s look at the answer choices:

Answer Choice (A) We need the classes to increase in enrollment because otherwise the number of students would remain constant while the number of teachers would increase. This would throw off our proportion so the average number of students per teacher would not remain the same.

Correct Answer Choice (B) This is exactly what we need. If the number of teachers goes up, we need the number of students to increase as well in order for the average number of students per teacher to remain the same.

Answer Choice (C) This is unsupported. We know that the increase in budget allowed the school district to hire more teachers, but it’s really immaterial how much the budget increased by. We already know the number of teachers increased, how the budget corresponds to that is not necessary for us to understand.

Answer Choice (D) There’s nothing to suggest that the district either retained old teachers or hired new teachers–the bottom line is that the number of teachers increased.

Answer Choice (E) This is completely unrelated to the ratio of students to teachers and is wholly unsupported by our passage.

Take PrepTest

Review Results

Leave a Reply